Chronology Current Month Current Thread Current Date
[Year List] [Month List (current year)] [Date Index] [Thread Index] [Thread Prev] [Thread Next] [Date Prev] [Date Next]

Re: [Phys-l] Ampere's original magnetic force law



On 04/14/2010 02:46 AM, Bob Sciamanda wrote:
I have been asked (by engineers at Penn State) to submit this topic
to the List for comments:

http://www.df.lth.se/~snorkelf/LongitudinalMSc.pdf

It concerns a controversy regarding Ampere's original formulation of
the basic magnetic force law between current elements. Apparently he
was persuaded to this version because it obeys N3 even in the strong
form (equal and opposite "central" forces). This is contrary to the
traditional force law, which comes from the Biot Savart relation and
the Lorentz QVxB force.

Examples of experimental evidence for the Ampere longitudinal force
component are cited. Should these results be attributed to an
other-than-magnetic component of the force between current elements?

Could the original Ampere magnetic force law possibly be reconciled
with Maxwell's equations and the Lorentz force?

Your comments and analyses, please.

We have a perfectly good well-tested theory of classical
electromagnetism, namely the Maxwell equation and the Lorentz
force law.

It involves charges (including moving charges) and fields,
plus the interaction between charges and fields.

The Maxwell/Lorentz theory does not "come from" the Biot-Savart
law but rather the other way around; the theory produces the
Biot-Savart law as a limiting case subject to some simplifying
approximations.

The theory is not based on "the force between current elements"
although the theory can handle that as a limiting case subject
to some simplifying assumptions. Foundational questions (such
as the existence or non-existence of "longitudinal" forces)
should not be asked or answered in terms of "the force between
current elements".

The theory is relativistically correct and can be written
in manifestly invariant form, correct to all orders in v/c.
The electromagnetic field tensor is the same in all frames.
In any particular frame, we can identify a component of the
EM field tensor that we call the "electric" field and another
component that we call the "magnetic" field. These components
are not separately invariant; the thing we call the electric
field in one frame is partly magnetic in another frame, and
vice versa.

In some sense magnetostatics can be seen as the first-order
correction to electrostatics, valid (and obligatory!) to
first order in v/c.

There are plenty of "other-than-magnetic" forces in ordinary
circuits.
-- There are the forces of constraint that cause the
current to remain inside the wire, to follow the wire
around corners, et cetera. These forces involve
electrostatics, plus quantum-mechanical solid-state
physics (the band structure of metals, the nature of
insulators, et cetera).
-- There are higher-than-first-order relativistic terms
(if you choose to expand things in powers of v/c).
-- There is energy and momentum in the fields, not just
in the wires.

It must be emphasized that the conventional equations of
electromagnetism can be formulated in a way that is correct
to all orders in v/c. It is only the notion of separate
"magnetic-only" phenomena that is limited to first order
in v/c.

The higher-order relativistic effects are not large
enough to be significant in the experimental situations
considered by the thesis ... as recognized in section 3.4
of the thesis.

==============

1) I am not impressed by the experimental data cited in the
thesis.

All of the experiments involving liquid conductors can be
explained in terms of the conventional Maxwell/Lorentz
equations (which predict a pinch effect) plus the usual
equations of fluid dynamics.

All of the data involving disks and shattered wires can
probably be explained in terms of nonlinear localized
heating, plus pinch effects, plus arc/vapor effects.

2) There is a glaring omission from the theoretical
analysis in the thesis.

Things have progressed quite a bit since "Ampère's original
magnetic force law". In particular, Maxwell emphasized the
idea that current "A" produces a field that acts on current
"B" ... as opposed to an action-at-a-distance theory (such
as Ampère's theory) where current acts on current without
bothering with a field.

If we think there should be a field, the humongous open
question is: Can you tell me the value -- or even the
direction -- of the field associated with the "longitudinal
electrodynamic forces" that are the subject of discussion?
The thesis is restricted to the current/current interaction,
without specifying the field. I cannot imagine any way of
calculating the field that would be consistent with the
geometrical symmetries of spacetime. So unless you want
to overthrow special relativity (as well as Maxwell/Lorentz
electromagnetism) there is a fundamental problem here.

=========

As James Randi is fond of saying, extraordinary claims
require extraordinary proof.

If you want me to take seriously the claims of extraordinary
longitudinal forces, I need to see, at a minimum:
a) Some experimental data that controls for the obvious
non-extraordinary explanations of the observations; and
b) A relativistically-correct theoretical description of
the forces.